Phương pháp dồn biến giải bất đẳng thức

60 95 0
Phương pháp dồn biến giải bất đẳng thức

Đang tải... (xem toàn văn)

Tài liệu hạn chế xem trước, để xem đầy đủ mời bạn chọn Tải xuống

Thông tin tài liệu

PHƯƠNG PHÁP DỒN BIẾN Phan Thành Việt Nội dung: Giới thiệu BĐT biến với cực trò đạt đối xứng Dồn biến kó thuật hàm số BĐT biến với cực trò đạt biên BĐT biến Dồn biến hàm lồi Dồn biến giá trò trung bình Đònh lý dồn biến tổng quát Nhìn lại 10 Bài tập Giới thiệu Các bạn thân mến, nhiều số BĐT mà ta gặp có dấu đẳng thức biến số Một ví dụ kinh điển √ Ví dụ 1: (BĐT Cauchy) Cho x, y, z > x + y + z ≥ 3 xyz Có thể nói số lượng BĐT nhiều nhiều bạn thấy điều hiển nhiên Tất nhiên, không hẳn Tuy nhiên, trường hợp đẳng thức không xảy tất biến ta lại thường rơi vào trường hợp khác, tổng quát hơn: có số (thay tất cả) biến Ở dẫn ví dụ chứng minh phần sau Ví dụ 2: (VMO) Cho x, y, z ∈ R, x2 + y + z = Thì 2(x + y + z) − xyz ≤ 10 vò) Trong BĐT dấu "=" xaûy x = y = 2, z = −1 (và hoán Có thể nhiều bạn ngạc nhiên biết có bất đẳng thức mà dấu "=" xảy biến khác Ví dụ sau chứng minh phần sau Ví dụ 3: (Jackgarfukel) Cho a, b, c số thực không âm có tối đa số Thì ta coù: a 5√ b c √ ≤ a+b+c +√ +√ c+a a+b b+c Ở đây, dấu đẳng thức xảy a = 3b > 0, c = (và dạng hoán vò) Các bạn tự hỏi giá trò chẳng hạn (3, 1, 0) có đặc biệt mà làm cho đẳng thức xảy Một cách trực giác, ta thấy dường điểm đặc biệt có biến Vì giả thiết biến không âm, nên biến gọi biến có giá trò biên Tóm lại, BĐT mà ta gặp, có trường hợp dấu "=" xảy thường gặp: trường hợp tất biến (ta gọi "cực trò đạt tâm"), tổng quát trường hợp có số biến (ta gọi "cực trò đạt có tính đối xứng"), trường hợp khác dấu "=" xảy có biến có giá trò biên (và ta gọi "cực trò đạt biên") Phương pháp dồn biến đặt để giải BĐT có dạng Ý tưởng chung là: ta đưa trường hợp có hai biến nhau, biến có giá trò biên, số biến giảm Do BĐT đơn giản BĐT ban đầu, đặc biệt BĐT biến cách khảo sát hàm biến số ta chứng minh BĐT đơn giản Chính tư tưởng giảm dần số biến nên phương pháp gọi phương pháp dồn biến Bây trình bày kó thuật phương pháp thông qua toán cụ thể Đối tượng quan trọng mà muốn bạn đọc nắm bắt BĐT với biến số Sau đó, mở rộng cho biến trình bày Cuối cùng, đến với phương pháp dồn biến tổng quát cho n biến số, bạn đọc từ kết "cổ điển" tới cải tiến nhỏ sau kết tổng quát Tinh thần xuyên suốt muốn bạn đọc cảm nhận tính tự nhiên vấn đề Qua đó, bạn lý giải "tại sao", để tự bước đường sáng tạo *Ghi chú: Chúng đánh dấu toán theo mục Vì số lượng đònh lý nên không đánh dấu Chúng cố gắng ghi tên tác giả nguồn trích dẫn tất kết quan trọng, ngoại trừ kết BĐT biến với cực trò đạt đối xứng Xin phác họa lại tư tưởng sau Bài toán có dạng f (x, y, z) ≥ với x, y, z biến số thực thỏa mãn tính chất Điều mong muốn có đánh giá f (x, y, z) ≥ f (t, t, z) với t đại lượng thích hợp tùy theo liên hệ x, y, z (ta gọi kó thuật dồn biến nhau) Sau kiểm tra f (t, t, z) ≥ để hoàn tất chứng minh Lưu ý biến chuẩn hóa bước cuối toán với biến Trong mục này, xem xét ví dụ Bài toán (BĐT Cauchy) Cho x, y, z > 0, chứng minh √ x + y + z ≥ 3 xyz Lời giải: Vì BĐT đồng bậc nên cách chuẩn hóa ta giả sử x+y+z = (*) Viết lại toán dạng f (x, y, z) ≥ với f (x, y, z) = − 27xyz Ta điều kiện (*) bảo toàn (tức thấy thay x y t = x+y có t + t + z = 1), nên ta phải xem xét thay đổi xyz Theo BĐT Cauchy với biến (chứng minh đơn giản) xy ≤ t2, nên xyz ≤ t2z Vậy f (x, y, z) ≥ f (t, t, z) Cuối để ý z = − 2t nên ta coù: f (t, t, z) = − 27t2 z = − 27t2 (1 − 2t) = (1 + 6t)(1 − 3t)2 ≥ toán chứng minh xong Đẳng thức xảy x = y 3t = 1, nghóa x = y = 1/3, tương đương với x = y = z *Nhận xét: 1) Có thể nhiều bạn bỡ ngỡ với cách chuẩn hóa Chúng xin nói rõ: bí ẩn Nếu thích, bạn hoàn toàn chuẩn hóa theo cách khác, chẳng hạn giả sử xyz = chứng minh f (x, y, z) ≥ với f (x, y, z) = x + y + z − Khi bước dồn biến chứng √ minh f (x, y, z) ≥ f (t, t, z) với t = xy Đề nghò bạn đọc tự lý giải √ lại xét t = xy, sau lời giải ta xét t = x+y hoàn thành chứng minh theo cách 2) Bạn đọc thắc mắc: không cần chuẩn hóa không? Câu trả lời là: được! Thật vậy, hoàn toàn xét toán f (x, y, z) ≥ √ với f (x, y, z) = x + y + z − xyz Khi bước dồn biến chứng minh √ hay t = xy Thực chất, điều f (x, y, z) ≥ f (t, t, z) với t = x+y hoàn toàn dễ hiểu, tương ứng BĐT có điều kiện BĐT không điều kiện (qua kó thuật chuẩn hóa) 3) Chúng nghó bạn đồng ý rằng: toán chuẩn hóa (tức BĐT có điều kiện) "gợi ý" cho cách dồn biến (phải đảm bảo điều kiện), nhiên, ngược lại toán chưa chuẩn hóa (BĐT không điều kiện) có nhiều cách để dồn biến (nói chung, ta chọn cách dồn biến cho bảo toàn "nhiều" biểu thức BĐT - điều tương đương với chuẩn hóa cho biểu thức có dạng đơn giản nhất) Do đó, phối hợp tốt kó thuật chuẩn hóa dồn biến điều cần thiết Tuy nhiên, quen với điều bạn thấy khác biệt đáng kể chúng Bài toán (BĐT Schur) Cho a, b, c ≥ 0, chứng minh rằng: a3 + b3 + c3 + 3abc ≥ a2 (b + c) + b2 (c + a) + c2(a + b) Lời giải: Xét f (a, b, c) = a3 + b3 + c3 + 3abc − a2(b + c) − b2 (c + a) − c2 (a + b) Đặt t = b+c , ta hi voïng: f (a, b, c) ≥ f (a, t, t) Xeùt d = f (a, b, c) − f (a, t, t) = b + c − a (b − c)2 Ta thấy với a, b, c số không âm tùy ý d ≥ Tuy nhiên, giả sử a = min{a, b, c} ta có d ≥ Khi ta phải chứng minh f (a, t, t) ≥ Nhưng BĐT tương đương với a(a − t)2 ≥ nên hiển nhiên Bài toán chứng minh xong *Nhận xét: Việc giả sử a = min{a, b, c} thủ thuật thường áp dụng để dồn biến Nhắc lại BĐT biến đối xứng ta giả sử a ≤ b ≤ c (hoặc a ≥ b ≥ c), trường hợp BĐT biến hoán vò vòng quanh ta giả sử a = min{a, b, c} (hoặc a = max{a, b, c}) Bài toán Cho a, b, c số thực dương có tích Chứng minh raèng: 1 + + + ≥ a b c a+b+c Hướng dẫn: Nếu toán ban đầu toán quen thuộc, toán khó Với kinh nghiệm thu từ toán 1, nghó tới việc dồn biến theo trung bình nhân để khai thác giả thiết tích ba số Một lời giải theo hướng bạn Yptsoi (Đài Loan) đưa lên diễn đàn Mathlinks, mà sau xin dẫn lại cách vắn tắt √ √ Ta chứng minh f (a, b, c) ≥√f (a,√ bc, bc) giả sử a ≥ b ≥ c Tiếp theo, ta chứng minh f (a, bc, bc) ≥ 5, hay laø √ f , x, x ≥ 5, vớ i x = bc x2 BĐT tương đương với (x − 1)2 (2x4 + 4x3 − 4x2 − x + 2) ≥ Vì biểu thức ngoặc thứ hai dương với x > nên chứng minh hoàn tất Đẳng thức xảy a = b = c = Qua ví dụ trên, thấy cách dồn biến trung bình cộng trung bình nhân thật hữu dụng Tuy nhiên, cách dồn biến vô phong phú uyển chuyển Ví dụ sau minh họa cho điều Bài toán 4.(Iran 1996) Chứng minh với a, b, c > thì: (ab + bc + ca) 1 + + ≥ (a + b)2 (b + c)2 (c + a)2 Hướng dẫn: Đây toán khó Các bạn thấy điều qua kiện dấu "=" đạt a = b = c có a = b, c → Các bạn nên thử để thấy cách dồn biến thông thường trung bình cộng trung bình nhân dẫn đến BĐT vô phức tạp Lời giải sau lấy từ ý thầy Trần Nam Dũng, mà nhìn kó bạn thấy mối tương quan, không tính toán mà tư duy, kó thuật chuẩn hóa dồn biến, mà đề cập nhận xét 3) toán Vì BĐT đồng bậc nên ta giả sử ab + bc + ca = (*) Bây ta hi vọng có đánh giá f (a, b, c) ≥ 94 với f (a, b, c) biểu thức thứ hai vế trái BĐT cần chứng minh Ở t phải thỏa liên hệ (*), nghóa t2 + 2tc = Bằng cách giả sử c = min{a, b, c} ta chứng minh f (a, b, c) ≥ f (t, t, c) Cuối cùng, ta kieåm tra f (t, t, c) ≥ 94 Ở bạn đọc thay vào BĐT để thấy: c = 1−t 2t f (t, t, c) = (1 − t2)(1 − 3t2)2 ≥0 4t2(1 + t2) Bài toán chứng minh xong! *Nhận xét: Ở bước cuối, bạn không chuẩn hóa mà quay lại BĐT đồng bậc: + ) ≥ (t + c)2 4t2 2 ⇔ (t + 2tc)(8t + (t + c) ) − 9(t + c)2 t2 ≥ ⇔ 2tc(t − c)2 ≥ (t2 + 2tc)( Cuối đến với ví dụ mà cực trò không đạt tâm, BĐT đối xứng Các bạn thấy rằng, đường phần quan trọng dồn hai biến nhau, sau cực trò đạt tâm hay không điều mấu chốt Bài toán (VMO) Cho x, y, z số thực thỏa mãn: x2 + y + z = Chứng minh raèng: 2(x + y + z) − xyz ≤ 10 Lời giải Đặt f (x, y, z) = 2(x + y + z) − xyz Chúng ta hi vọng coù f (x, y, z) ≥ f (x, t, t), t2 = (y + z 2)/2 (*) (chúng nghó bạn đọc tự lý giải điều này) Lưu ý (*) t nhận giá trò, để đònh ý ta xét t ≥ Ta có: d = f (x, y, z) − f (x, t, t) = 2(y + z − 2t) − x(yz − t2) Ta thaáy y + z − 2t ≤ yz − t2 ≤ Do để có d ≤ ta cần x ≤ Từ đó, ta giả sử x = min{x, y, z} Xét trường hợp x ≤ Khi ta dồn biến phải chứng minh f (x, t, t) ≤ 10 Thay t = (9 − x2 )/2 ta coù: g(x) = f (x, t, t) = 2x + Ta coù: g (x) = 2(9 − x2) − x(9 − x2)/2 4x 3x2 − −√ 2 18 − 2x2 Giải ta thấy phương trình g (x) = có nghiệm âm x = −1 Hơn g liên tục g (−2) > > g(0) nên suy g đổi dấu từ dương sang âm qua điểm x = −1 Vậy ∀x ≤ g(x) ≤ g(−1) = 10 ta có điều phải chứng minh Trường hợp đẳng thức đạt x = −1, y = z = Phần lại ta phải giải trường hợp x > 0, tức số x, y, z dương Lúc dấu BĐT thực ta cần đánh giá đơn giản thông qua dồn biến Nếu x ≥ 3/4 f (x, y, z) = 2(x + y + z) − xyz ≤ √ 27 < 10 3(x2 + y + z ) − ( )3 = 27− 64 Nếu x ≤ 3/4 f (x, y, z) = 2(x + y + z) − xyz ≤ 2( √ 2(y + z )+3/4) ≤= 2( 18 +3/4) < 10 Bài toán chứng minh xong! Dồn biến kó thuật hàm số Đây kó thuật quan trọng phương pháp dồn biến Tuy nhiên giới thiệu sau phần nhằm trang bò cho bạn kó thuật cần thiết trước qua mục sau Hơn nữa, nghó quen với bạn phân biệt cực trò đạt tâm hay biên, mục nhẹ nhàng Trong $2 thấy để chứng tỏ f (x, y, z) ≥ f (t, t, z) ta việc xét hiệu d = f (x, y, z) − f (t, t, z) tìm cách đánh giá cho d ≥ Tuy nhiên, dạng BĐT đơn giản, phù hợp với biến đổi đại số Giả sử ta phải làm việc với biểu thức f có dạng, chẳng hạn, như: f (x, y, z) = xk + y k + z k với k > cách biến đổi đại số trở nên cồng kềnh phức tạp Kó thuật hàm số dùng để giải trường hợp Ý tưởng này, chẳng hạn để chứng minh f (x, y, z) ≥ f (x, t, t) với t = (y + z)/2, ta xét hàm: g(s) = f (x, t + s, t − s) với s ≥ Sau chứng minh g tăng với s ≥ (thông thường dùng công cụ đạo hàm tiện lợi), suy g(s) ≥ g(0), ∀s ≥ 0, ta thu điều mong muốn Một ví dụ quen thuộc với bạn dồn biến hàm lồi, nhiên quan sát kó thuật dồn biến bối cảnh tổng quát hơn, vấn đề hàm lồi trở lại mục sau toán với n biến Chúng nhấn mạnh rằng, kó thuật khó, chứa đựng nét tinh tế phương pháp dồn biến Những ví dụ sau thể rõ vẻ đẹp sức mạnh phương pháp dồn biến Bài toán Cho k > vaø a, b, c laø số không âm có tối đa số Chứng minh rằng: ( a k b k c k ) +( ) +( ) ≥ min{2, k } b+c c+a a+b (∗) Lời giải: Tất nhiên ta cần chứng minh BĐT = 23k ⇔ k = ln3 − (caùc ln2 bạn suy nghó BĐT cho trường hợp lại dẫn đến BĐT cho trường hợp tổng quát) Chú ý với k đẳng thức xảy hai chỗ a = b = c a = b, c = (và hoán vò) Không tổng quát giả sử a + b + c = b ≥ c ≥ a Đặt t = b+c m = b−c , suy b = t + m, c = t − m, a = − 2t Khi vế trái 2 BĐT cần chứng minh laø: f (m) = − 2t 2t k + t+m 1−t−m k + t−m 1+m−t k Vì c ≥ a neân 3t − ≥ m ≥ 0, ≥ b + c = 2t nên 12 ≥ t ≥ 13 Ta khảo sát f (m) miền m ∈ [0, 3t − 1] với t ∈ [ 13 , 12 ] số Ta coù: k(t + m)k−1 k(t − m)k−1 − (1 − t − m)k+1 (1 + m − t)k+1 (t + m)k−1 (t − m)k−1 f (m) ≥ ⇔ ≥ (1 − t − m)k+1 (1 + m − t)k+1 k+1 [ln(1 − t − m) − ln(1 + m − t)] ≥ ⇔ g(m) := [ln(t − m) − ln(t + m)] − 1−k f (m) = Tiếp tục khảo sát g, ta có: k+1 1 + + + ≥0 t−m t+m 1−k 1−t−m 1+m−t k+1 2(1 − t) −2t + ≥ (1) ⇔ (t − m)(t + m) − k (1 − t − m)(1 + m − t) g (m) = − Đánh giá k+1 1−k ≥ 2, để chứng minh (1) ta cần chứng minh 2(1 − t) −t + ≥ (1) −m (1 − t)2 − m2 ⇔ u(m) = −t + 4t2 − 3t3 + 3tm2 − 2m2 ≥ ⇔ t2 Thật vậy, u (m) < nên u(m) ≥ u(3t − 1) = 2(3t − 1)(2t − 1)2 ≥ Vậy g(m) đồng biến suy g(m) ≥ g(0) = suy f (m) ≥ suy f (m) ≥ f (0) Nhớ m = b = c = t Cuối cùng, ta cần chứng minh h(t) := f (0) ≥ Viết laïi: h(t) = k − 2t 2t +2 t 1−t k Ta khảo sát h(t) miền t ∈ [0, 13 ] Ta coù: 2ktk−1 k (1 − 2t)k−1 − ≤0 h (t) = (1 − t)k+1 2k tk+1 ⇔ 2k+1 t2k ≤ [(1 − t)(1 − 2t)]k−1 (2) Trong BĐT cuối, vế trái hàm đồng biến theo t vế phải hàm nghòch biến theo t, lưu ý t ≤ 13 nên để chứng minh (2) ta cần: 2k+1 2k ≤ [(1 − )(1 − )]k−1 3 Bất đẳng thức đúng, nên h(t) nghòch biến, suy h(t) ≥ h( ) = Baøi toán giải trọn vẹn! Nhận xét: Để thấy nét đẹp toán này, xin dẫn số trường hợp riêng nó, thân chúng toán hay biết đến cách rộng rãi 1) Trường hợp k = 1, ta thu BĐT Netbit: a b c + + ≥ b+c c+a a+b Đây BĐT tiếng Một cách chứng minh "kinh điển" là: b c a+b+c a+b+c a+b+c a + + +3= + + b+c c+a a+b b+c a+c a+b 1 + + ) = (a + b + c)( b+c c+a a+b 9 ≥ (a + b + c) = (b + c) + (c + a) + (a + b) 2) Trường hợp k = 12 , ta thu BĐT sau: a + b+c b + c+a c ≥2 a+b Đây toán đẹp, trước biết đến BĐT ngược chiều với BĐT Netbit Có lời giải đơn gản, dùng BĐT Cauchy: 2a 2a a = ≥ b+c a+b+c a(b + c) 3) Trường hợp k ≥ 23 , ta có BĐT sau: ( a k a k a k ) +( ) +( ) ≥ k b+c b+c b+c 10 Đònh nghóa 1: • Không gian Rn tập hợp thứ tự x = (x1, x2 , , xn) với xi ∈ R, ∀i • Một dãy {xm = (x1,m , , xn,m)} Rn gọi hội tụ z = (z1, , zn) ∈ Rn dãy xi,m hội tụ zi m → ∞, ∀i = 1, 2, , n • Cho D ⊂ Rn Một hàm số f : D → R gọi liên tục D nếu: với dãy {xm} ⊂ D với z ∈ D cho {xm } hội tụ z, ta có: f (xm ) hội tụvề f (z) Đònh nghóa 2: Cho D ⊂ Rn Ta nói: • D đóng với dãy {xm } ⊂ D với z ∈ Rn cho {xm } hội tụ z, ta có z ∈ D • D bò chặn tồn số thực M cho: ∀x = (x1, , xn) ∈ D}, |xi | ≤ M, ∀i = 1, 2, , n Ví dụ tập hợp hữu hạn đóng bò chặn Xuất phát điểm kết tuyệt đẹp sau đây: Đònh lý 1: Cho D đóng bò chặn Rn , f : D → R liên tục Thì f đạt giá trò nhỏ D, nghóa tồn x0 ∈ D cho: f (x0 ) ≤ f (x), ∀x ∈ R Đây kết có chương trình phổ thông nước, nhiên nước ta xem thuộc "Toán cao cấp" Tuy nhiên, để tiện lợi cho bạn đọc dẫn chứng minh mà bạn hoàn toàn hiểu với kiến thức phổ thông Chúng nhắc lại kết có SGK: " dãy số thực đơn điệu bò chặn hội tụ" "Tiên đề" sử dụng để chứng minh kết dãy n Đònh nghóa 2: Cho dãy số {am }∞ m=1 (trong R R ) Một dãy ∞ ∞ {amk }∞ k=1 gọi dãy dãy {am }m=1 {mk }k=1 dãy tăng ngặt số nguyên dương ∞ *Ví dụ: {a2m }∞ m=1 dãy dãy {am }m=1 Dưới cận số bỏ qua không gây hiểu lầm Bổ đề 1: (Weierstrass) Mỗi dãy am bò chặn R có dãy hội tụ Chứng minh: Ta chứng minh có dãy đơn điệu xong Xét tập T := {m ∈ Z + |∃m > m cho am ≥ am } Neáu T hữu hạn 46 dãy {am } giảm kể từ số Nếu T vô hạn ta trích dãy tăng Trong hai trường hợp ta có dãy đơn điệu Bổ đề 2: (Weierstrass) Mỗi dãy am bò chặn Rn có dãy hội tụ Chứng minh: Xét {am = (x1,m, , xn,m)} dãy bò chặn Rn Khi dãy {x1,m} bò chặn R nên có dãy {x1,mk1 } hội tụ Dãy {x2,mk1 } bò chặn R nên có dãy {x2,mk2 } hội tụ Bằng cách lấy "dãy dãy con" liên tiếp vậy, cuối ta thu dãy {amk = (x1,mk , , xn,mk } maø ∀i = 1, 2, , n, ta có dãy {xi,mk } hội tụ R Điều có nghóa dãy {amk } hội tụ Rn Bổ đề 3: (Tính đầy đủ R) Cho A tập bò chặn R Thì tồn M ∈ R cho: M ≤ A (nghóa M ≤ a, ∀a ∈ A) có dãy {ak } A hội tụ M Ta kí hiệu M = inf A Chứng minh: Ta chứng minh ∀ε > 0, ∃a ∈ A, a − ε ≤ A Giả sử ngược lại Khi lấy x1 ∈ A tùy ý, quy nạp ta xây dựng daõy {xm } A cho xm+1 ≤ xm − ε, ∀m ∈ Z + Khi ta coù: xm ≤ x1 − (m − 1)ε, ∀m ∈ Z + điều mâu thuẫn với A bò chận Như vậy, ∀m ∈ Z + , tồn taïi am ∈ A cho am − m1 ≤ A Vì dãy {am } bò chặn nên có dãy {amk } hội tụ M R Ta chứng minh M ≤ A xong Thật vậy, lấy a ∈ A amk − m1k ≤ a, ∀k ∈ Z + , neân cho k → ∞ suy M ≤ a Chứng minh đònh lý 1: Xét A = f (D) Ta chứng minh A có phần tử nhỏ Ta A có tính chất sau: dãy {am } chứa A am → α α ∈ A Thật vậy, theo đònh nghóa ta có xm ∈ D cho f (xm ) = am → α Vì dãy {xm } bò chặn (chứa D) nên có dãy {xmk } hội tụ c Rn Vì D đóng nên c ∈ D Vì f (xm ) → α nên Vì f (xmk ) → α Mặt khác, {xmk } → c f liên tục nên f (xmk ) → f (c) Vì giới hạn nên f (c) = α Bây giờ, ta thấy A bò chặn (vì từ lập luận với α = −∞ ta gặp mâu thuẫn) Do tồn M = inf A Do đònh nghóa inf tính chất A vừa trên, suy M ∈ A Vậy A có phần tử nhỏ M Đònh lý chứng minh xong! 47 Đònh lý mở rộng kết quen thuộc có SGK: "Cho [a, b] khoảng đóng R f : [a, b] → R liên tục, f có giá trò nhỏ [a, b]" Do đó, mặt trực giác đònh lý rõ ràng Tuy nhiên, bạn khó hình dung đònh lý liên quan đến vấn đề dồn biến? Hệ sau đònh lý "chìa khóa" cho đònh lý dồn biến Lưu ý tất kết mục không cần điều kiện f đối xứng Đònh lý 2: Cho: • D tập đóng, bò chận Rn , Λ tập đóng D • T : D → D phép biến đổi • f : D → R hàm số liên tục thỏa mãn f (x) > f (T (x)), ∀x ∈ D\Λ Thì ta có GTNN f đạt Λ, nghóa là: f (x) > min{f (y)}, ∀x ∈ D\Λ y∈Λ Chứng minh: Do đònh lý 1, tồn x0 ∈ D cho f (x0 ) ≤ f (x), ∀x ∈ D Nếu x0 không thuộc Λ f (x0 ) > f (T (x0)), mâu thuẫn Vậy x0 ∈ Λ ta có điều phải chứng minh *Ghi chú: Ta thấy phép biến đổi T : D → D thực đònh lý đòi hỏi tính chất T D\Λ Do với x ∈ Λ T (x) lấy giá trò tùy ý ta xem T (x) = x Quy ước sử dụng phần lại, nghóa T (x) = x, ∀x ∈ Λ ta quan tâm giá trò T D\Λ Đây hệ đơn giản phải không bạn, nhiên ý tưởng dồn biến lộ rõ Để minh họa, dẫn chứng minh cho BĐT Cauchy Bài toán 1: (BĐT Cauchy) Cho n số thực không âm x1 , , xn Chứng minh raèng: √ x1 + + xn ≥ n n x1 xn Chứng minh: Bằng cách chuẩn hóa, ta giả sử x1 xn = chứng minh x1 + + xn ≥ n Tất nhiên ta cần xét xi ≤ n, ∀i Xét: D = {x = (x1 , , xn)|xi ∈ [0, n], x1 xn = 1} dễ thấy D đóng bò chặn Xét Λ = {x0 = (1, 1, 1, , 1)} Xét f : D → R liên tục sau: với x = (x1, , xn) ∈ D 48 f (x) = x1 + + xn Xét T : D\Λ → D sau: Với x = (x1, , xn) ∈ D\Λ, tồn xi = xj ta đặt T (x) thu từ x sau thay xi xj trung bình nhân chúng, dễ thấy √ √ f (x) − f (T (x)) = ( xi − xj )2 > Vaäy ta áp dụng đònh lí để suy f (x) ≥ f (x0), ∀x ∈ D, hôn dấu ” = ” xảy x = x0 Trong nhiều trường hợp, hàm f không đủ tốt ta có có điều kiện f (x) ≥ f (T (x)) Tất nhiên ta áp dụng đònh lý Một đòi hỏi hợp lý phép biến đổi T phải đủ tốt để bù lại (nhớ phép biến đổi T ta chọn) Điều đưa đến: Đònh lý 3: Cho: • D tập đóng, bò chận Rn , Λ tập đóng D • T : D → D phép biến đổi cho tồn hàm số h liên tục D → R thỏa mãn: h(T (x)) < h(x),∀x ∈ D\Λ • f : D → R hàm số liên tục thỏa mãn f (x) ≥ f (T (x))∀x ∈ D Thì ta có GTNN f D GTNN f Λ, nghóa là: f (x) ≥ min{f (y)}, ∀x ∈ D y∈Λ Mặc dù trường hợp riêng đònh lý tổng quát cuối bài, tầm quan trọng đònh lý nên dẫn chứng minh cho Chứng minh: Lấy y0 ∈ Λ cho f (y0 ) = min{f (y)} Giả sử phản chứng tồn y∈Λ z ∈ D cho f (z) < f (y0 ) Tất nhiên ta giả sử h(x) ≥ 0, ∀x ∈ D (nếu không việc thay h h = h − M, với M GTNN h D) Chọn ε > đủ nhỏ ta có: f (z) + εh(z) < f (y0 ) Đặt g(x) := f (x) + εh(x), ∀x ∈ D Thì g : D → R liên tục, g(x) > g(T (x))∀x ∈ D\Λ g(z) < f (y0 ) ≤ min{g(y)} Điều mâu thuẫn với đònh lý y∈Λ Sau hệ ấn tượng đònh lý Hệ 1: (SMV-Strongly Mixing Variables) Cho: • D ∈ Rn , D = {x = (x1, , xn)|xi ≥ α, xi = ns = const} vaø s0 := (s, s, , s) ∈ D • Phép biến đổi T : D → D sau: với phần tử a = (a1, , an) ∈ D, a = s0 , ta choïn số i = j (tùy theo hàm f bên dưới) cho 49 = aj , thay , aj bới trung bình cộng chúng • f : D → R hàm số liên tục thỏa mãn: f (a) ≥ f (T (a)), ∀a ∈ D Khi đó: f (a) ≥ f (s0 ), ∀a ∈ D n Chứng minh: Với phép biến đổi T vậy, ta chọn h(x) = i=1 x2i , ∀x = (x1, , xn) ∈ D Áp dụng đònh lý (ở Λ = {s0 }) *Nhận xét: Thông thường, áp dụng ta lấy , aj max {a1, , an} Khi đó, chứng minh từ phần tử D, sau vô hạn lần lặp T thu (s, s, , s), sử dụng tính liên tục f ta thu kết luận Riêng trường hợp (min max) không thiết thay , aj trung bình cộng mà tổng quát hơn: Hệ 2: Cho: • D ∈ Rn đóng bò chặn Gọi Λ tập hợp phần tử D có dạng (s, s, , s), giả sử Λ khác rỗng • Phép biến đổi T : D → D sau: với phần tử a = (a1, , an) ∈ D\Λ, ta chọn số i = j cho , aj max {a1, , an}, sau thay , aj α, β ∈ (ai , aj ) • f : D → R hàm số liên tục thỏa mãn: f (a) ≥ f (T (a)), ∀a ∈ D Khi đó: f (x) ≥ min{f (y)}, ∀x ∈ D y∈Λ Chứng minh: Một cách tự nhiên, ta hi vọng vào hàm h(a) = max{a1, , an} − min{a1, , an}, ∀a = (a1, , an) ∈ D Tuy nhiên, ta h(a) > h(T (a)), ∀a ∈ D\Λ Đó n số a1, , an có nhiều số max hay {a1, , an} Nhưng ta việc thay T T ∗ = T n (T k nghóa lặp lại T với k lần) h(a) > h(T ∗(a)), ∀a ∈ D\Λ ta áp dụng đònh lý Tuy nhiên, phép biến đổi T không đủ, ví dụ ta chưa biết xác dồn biến biên hay tâm Do đó, đònh lý mở rộng thành đònh lý dồn biến tổng quát sau Đònh lý 4: (GMV − General Mixing Variables) Cho: • D tập đóng, bò chận Rn , Λ tập đóng D • Tj : D → D phép biến đổi cho tồn hàm số hj liên tục D → R thỏa mãn: h(Tj (x)) < hj (x),∀x ∈ D\Λ, ∀j ∈ {1, , k} 50 • f : D → R liên tục thỏa mãn f (x) ≥ Thì f (x) ≥ min{f (y)}, ∀x ∈ D {f (Tj (x))}, ∀x ∈ D j∈{1, ,k} y∈Λ Ta sử dụng lại chứng minh đònh lý với cải tiến nhỏ Chứng minh: Lấy y0 ∈ Λ cho f (y0 ) = min{f (y)} Giả sử phản chứng tồn y∈Λ z ∈ D cho f (z) < f (y0 ) Tất nhiên ta giả sử hj (x) ≥ 0, ∀x ∈ D, ∀j = 1, , k Chọn ε > đủ nhỏ ta coù: f (z)+εhj (z) < f (y0 ), ∀j = 1, , k Đặt gj (x) := f (x) + εhj (x), ∀x ∈ D, ∀j = 1, , k Ñaët g(x) = min{g1(x), , gk (x)}, ∀x ∈ D Thì g : D → R liên tục, g(x) > g(T (x))∀x ∈ D\Λ vaø g(z) < f(y0) ≤ min{g(y)} Điều mâu y∈Λ thuẫn với đònh lý *Chi chú: Ta sử dụng kết gj hàm liên tục g = min{g1, , gk } hàm liên tục Tất nhiên ta cần chứng minh với k = 2, trường hợp cần để ý min{g1, g2 } = (g + g2 − |g1 − g2|) Còn kiện g(x) > g(T (x)), ∀x ∈ D\Λ rõ ràng, uj > vj , ∀j = 1, , k min{u1, , uk } > min{u1, , uk } Các bạn thân mến, hình thức phát biểu ngắn gọn GMV có tầm ứng dụng rộng rãi Cứ (hay vài) phép biến đổi T thích hợp ta lại có đònh lý dồn biến Chúng kết thúc mục hệ GMV, mà xem mở rộng SMV Hệ Cũng xin lưu ý kết có tên SMV UMV tổng quát so với đònh lý tên mà dẫn ban đầu Hệ 3: (UMV − Undefined Mixing Variables) Cho: • D ⊂ {x = (x1, , xn) ∈ Rn |xi ≥ 0, ∀i = 1, , n}, D đóng bò chặn Gọi Λ tập hợp phần tử D có t thành phần n − t thành phần (t ≥ 0) • phép biến đổi T1, T2 : D → D sau: với phần tử a = (a1, , an) ∈ D\Λ, chọn số i = j cho = min{at > 0, t = 1, , n} vaø aj = max{a1, , an}, sau thay , aj bới α, β ∈ (ai , aj ) (ứng với T1 ) α < < aj < β (ứng với T2) • f : D → R liên tục thỏa mãn: f (a) ≥ min{f (T1(a)), f(T1(a))}, ∀a ∈ D Thì f (x) ≥ min{f (y)}, ∀x ∈ D y∈Λ 51 Chứng minh: Choïn h1 (a) = max{a1, , an} − min{a1, , an} vaø h2 (a) = −h1 (a), ∀a = (a1, , an) ∈ D Tương tự hệ 2, ta thay T1 T1∗ = T1n T2∗ = T2n để có: h1 (a) > h1 (T1∗(a)), h2 (a) > h2(T2∗(a)), ∀a = (a1, , an) ∈ D AÙp dụng GMV ta có điều phải chứng minh Nhìn lại Các bạn thân mến, có lẽ lúc tạm dừng để nhìn lại hành trình vừa qua Như nói $6, dồn biến biết đến từ sớm thông qua hàm lồi dẫn đến kết tuyệt đẹp BĐT Jensen xem tiêu chuẩn để dồn biến tâm cách toàn cục Về kết này, bạn tìm đọc cách đầy đủ "Bất đẳng thức" tiếng nhà toán học Hardy − P olya − Littewood Trong trường hợp biến, có lẽ quen thuộc với bạn đọc BĐT lượng giác, chẳng hạn nhö: √ 3 sinA + sinB + sinC ≤ (1) cosA + cosB + cosC ≤ (2) với A, B, C cạnh tam giác BĐT (1) thu cách áp dụng BĐT Jensen cho hàm lồi BĐT (2) tinh tế hơn, hàm f (x) = −cosx có f (x) = cosx nên lồi [0, π] Do ta áp dụng BĐT Jensen cho biến A,B,C Tuy nhiên, ta giả sử A ≤ B ≤ C A, B ∈ [0, π] nên ta dồn biến A, B Sau toán biến trở nên đơn giản Như vậy, tư tưởng dồn biến dần lộ rõ Thay mong muốn có cách dồn biến toàn cục, hi vọng bước đơn giản toán cách giảm dần số biến Đây tư tưởng phương pháp dồn biến Trong trường hợp biến, sau thực động tác dồn biến (bất kể biến nhau, hay dồn biến biên, hay dồn biến giá trò trung bình) gần toán biến xem giải đơn giản Do đó, phép dồn biến không cần tác dụng với biến mà tận dụng thứ tự biến BĐT đối xứng 52 Các bạn thân mến, dành mục để khảo sát vấn đề dồn biến cho BĐT biến để bạn nắm tư tưởng phương pháp, liệt kê tất kó thuật cần thiết Chẳng hạn dồn biến BĐT lượng giác với BĐT tuyệt đẹp Jackgarfulkel (xem phần tập) thú vò Tuy nhiên nghó trình bày tất nhàm chán vô vò, nắm tư tưởng bạn áp dụng trường hợp khác Đọc xong phần BĐT biến, có lẽ bạn đọc có cảm giác BĐT chuyển trường hợp biến biến đạt giá trò biên Phải nói điều cho hầu hết BĐT mà gặp Tuy nhiên, sau cung cấp cho bạn ví dụ nằm "thông lệ" Trong ví dụ này, chí BĐT xét đa thức đối xứng biến Ví dụ lấy từ ý tưởng anh Bùi Việt Anh Bài toán Cho a, b, c ≥ Khi BĐT: (a3 + b3 + c3 − 6abc)2 + ((a + b + c)3 − 36abc)2 ≥ xảy dấu "=" trường hợp (a, b, c) = (t, 2t, 3t), t ≥ (và hoán vò) Bạn đọc tự kiểm tra điều Như vậy, bạn yên tâm phương pháp dồn biến có ý nghóa Với toán biến thông thường phải thực lần động tác dồn biến nên phức tạp Trong trường hợp n biến tổng quát việc dồn biến trở nên khó khăn Ngoài BĐT Jensen cho phép dồn lúc n biến (nhưng đáng tiếc, giải lượng nhỏ BĐT) gần ta công cụ khác Trong trường hợp này, thông thường quy nạp ý hay Chúng dẫn ví dụ cho thấy tinh tế chứng minh quy nạp BĐT Bài toán (Phạm Kim Hùng) Cho n số thực dương a1 , a2, , an có tích Chứng minh với k > thì: 1 n + + + ≥ min{1, k } k k k (1 + a1 ) (1 + a2 ) (1 + an ) Baøi toán đưa lên Diễn Đàn Toán Học với tên gọi Thách Thức 1, khó Tuy nhiên, vô đơn giản ta 53 làm việc với toán tổng quát hơn: "Cho n số thực dương a1 , a2, , an có tích s ≥ Chứng minh với k > thì: 1 n √ }.” + + + ≥ min{1, k k k (1 + a1) (1 + a2) (1 + an ) 1+ ns Với toán tổng quát lại chứng minh quy nạp Thật vậy, xét toán với n số, ta giả sử an = min{a1, , an} Khi áp dụng giả thiết quy nạp cho (n − 1) số a1, a2, , an−1 có tích ≥ 1, ta đưa toán biến Công việc lại khảo sát hàm biến Một kó thuật khác để đưa BĐT n biến biến dồn biến giá trò trung binh $7 Như ra, ý tưởng cách dồn dựa cách dồn biến giá trò trung bình cho hàm lồi Đây cách dồn biến tốt có tính hữu hạn Tuy nhiên, áp dụng cho cực trò đạt tâm Bây ta phải đối mặt với khả cực trò đạt tâm biên Rõ ràng khả dồn biến không cao Do hi vọng vào điều tốt có cách dồn biến toàn cục, BĐT Jensen Với mục tiêu đó, đònh lý tuyệt đẹp phải kể đến đònh lý SMV (dồn biến mạnh) UMV (dồn biến không xác đònh) Hai đònh lý nói "anh em song sinh" SMV dùng để "chuyên trò" BĐT cực trò đạt tâm, cải tiến đáng kể không cần dồn biến mà cần dồn biến lớn biến nhỏ UMV đòi hỏi giả thiết đặt lên biến bất kì, nhiên cho phép ta dung hòa trường hợp cực trò đạt tâm biên dạng tổng quát Để cho hình thức đơn giản, đònh lý xét cho hàm đối xứng Chúng quan sát kết nhận thấy không cần thiết việc tách rời trường hợp, tìm kết hội tụ đầy đủ ưu điểm đònh lý Tuy nhiên trường hợp riêng Hệ $8 Đònh lý GMV không đơn tổng quát đònh lý kể trên, mà mở chân trời với kiểu dồn biến Một điều kì lạ đòi hỏi: x = (x1, x2, , xn) chưa rơi vào trường hợp "tới hạn" (tức thuộc Λ), thay (là T (x)) Nếu SMV ("cổ điển"), kiện dồn biến, lớn 54 nhỏ nhất, dẫn đến cảm nhận rõ ràng n biến tiến giá trò trung bình, trường hợp bổ đề dãy số không tác dụng Tuy nhiên, kết Các bạn thân mến, bạn hành trình, mà chọn tốt đầy đủ Có nhiều vấn đề không đưa ra, không trình bày kó, không coi trọng đầy đủ Cái mà coi trọng cố gắng để bạn thấy vấn đề cách nhanh chóng, rõ ràng hợp lý Hi vọng với tư tưởng mà khơi gợi bạn đủ cảm hứng khả để tiếp bước đường sáng tạo Cuối cùng, muốn gửi lời cảm ơn đặc biệt tới anh Phan Thành Nam anh Phạm Kim Hùng, người có nhiều kết ý tưởng sử dụng Chúng xin chân thành cảm ơn tất tác giả toán, nguồn trích dẫn, có thầy Phạm Văn Thuận − người cung cấp cho tài liệu dồn biến có giá trò 10 Bài tập Sau số tập dành cho bạn đọc Hi vọng bạn tìm nhiều niềm vui thử sức với chúng (ghi chú: Θ dễ, Φ trung bình, Ξ khó, Ψ cực khó) Θ Bài tập 1: (Asian Pacific Math.2004) Giả sử a, b, c số dương tùy ý Chứng minh BÑT (a2 + 2)(b2 + 2)(c2 + 2) ≥ 9(ab + bc + ca) Θ Bài tập 2: (MOSP 2001) Chứng minh a, b, c số dương có tích ta có BĐT (a + b)(b + c)(c + a) ≥ 4(a + b + c − 1) Θ Bài tập 3: Cho a, b, c không âm thỏa mãn a2 + b2 + c2 = Chứng minh a + b + c ≥ a2b2 + b2c2 + c2 a2 55 Θ Bài tập 4: (Huỳnh Tấn Châu) Cho x, y, z ≥ vaø x + y + z = 1.Chứng minh rằng: x3 + y + z + 6xyz ≥ Θ Bài tập 5: Chứng minh x, y, z số thực không âm thỏa mãn điều kiện x2 + y + z = ta có BĐT: 7(xy + yz + zx) ≤ 12 + 9xyz Φ Baøi tập 6: (Chọn đội tuyển Việt Nam 1996) Cho a, b, c số thực bất kì, chứng minh raèng: F (a, b, c) = (a + b)4 + (b + c)4 + (c + a)4 − (a4 + b4 + c4) ≥ Φ Bài tập 7: (Phạm Văn Thuận−Zhao Bin) Giả sử x, y, z ba số thực không âm có nhiều số Chứng minh x3 1 20 + + ≥ 3 +y y +z z +x (a + b + c)3 Φ Bài tập (Phạm Kim Hùng) Chứng minh với số thực a, b, c không âm ta có BĐT: 1 √ +√ +√ ≥ a+b+c 4a2 + bc 4b2 + ca 4c2 + ab Φ Bài tập 9: (Murray Klamkin) Chứng minh với số thực không âm a, b, c có tổng 2, (a2 + ab + b2)(b2 + bc + c2)(c2 + ca + a2) ≤ Ξ Bài tập 10: (Tổng quát RMO2000) Cho a, b, c ≥ vaø a + b + c = Tìm số k > nhỏ cho BĐT sau đúng: ak + bk + ck ≥ ab + bc + ca Ξ Bài tập 11: (Trung Quốc 2005) Cho a, b, c > ab + bc + ca = 1/3 56 Chứng minh raèng: a2 1 + + ≤3 − bc + b − ca + c − ab + Ξ Bài tập 12: (mathlinks) Cho a, b, c ≥ vaø ab + bc + ca = Chứng minh rằng: + a2 b2 + b2 c2 + c2 a2 + + ≥ 2 (a + b) (b + c) (c + a) Ξ Bài toán 13 Cho a, b, c ∈ [p, q] với < p ≤ q Tìm giá trò lớn của: b c a + + b+c c+a a+b Bài toán 14.(Jackgarfulkel) Cho tam giác nhọn ABC Chứng minh rằng: Φ a) A B C A B C sin + sin + sin ≥ (1 + sin sin sin ) 2 2 Φ b) A B C B C A cos + cos + cos ≥ √ (1 + cos cos cos ) 2 2 2 Φ Bài toán 15.(Jackgarfulkel) Cho tam giác ABC Chứng minh rằng: cos( A−B B−C C−A ) + cos( ) + cos( ) ≥ √ (sinA + sinB + sinC) 2 Ξ Baøi toán 16 (Phan Thành Nam) Cho ba sỗ thực x, y, z không âm có tổng Chứng minh raèng √ x + y + y + z + z + x2 ≥ Ξ Bài tập 17 (Vasile Cirtoaje) Xét ba số thực không âm a, b, c thỏa điều kiện a2 + b2 + c2 = Chứng minh rằng: 1 + + ≤ − ab − bc − ca 57 Ξ Bài tập 18: (Phan Thành Nam) Cho a, b, c ≥ thỏa mãn a + b + c = Chứng minh rằng: a) (VMEO1) a+ (b − c)2 + 12 b+ (c − a)2 + 12 c+ (a − b)2 √ ≤ 12 b) a + k(b − c)2 + Trong k = − a + k(b − c)2 + a + k(b − c)2 ≤ √ √ Ξ Bài toán 19 (Phan Thành Việt) Cho tam giác ABC có độ dài cạnh BC = a, CA = b, AB = c Goïi p nửa chu vi tam giác ma, mb , mc độ dài ba đường trung tuyến tương ứng hạ từ A, B, C xuống cạnh đối diện Chứng minh rằng: ma + mb + mc ≤ 3p2 + [(a − b)2 + (b − c)2 + (c − a)2] Bài tập 20: (Phan Thành Nam) Cho x, y, z ∈ [−1, 1] vaø x + y + z = Chứng minh Ξ a) √ + x + y + + y + z + + z + x2 ≥ Ψ b) + x + y2 + + y + z2 + + z + x2 ≥ Φ Bài tập 21 (Phạm Kim Hùng) Cho x, y, z, t ≥ vaø x + y + z + t = Chứng minh rằng: (1 + 3x)(1 + 3y)(1 + 3z)(1 + 3t) ≤ 125 + 131xyzt Φ Bài tập 22 (Bất đẳng thức Tukervici) Với số thực dương a, b, c, d a4 + b4 + c4 + d4 + 2abcd ≥ a2b2 + b2 c2 + c2d2 + d2 a2 + a2c2 + b2d2 58 Φ Bài tập 23 (Phạm Văn Thuận− Nguyễn Anh Tuấn) Xét số thực a, b, c, d thỏa mãn a2 + b2 + c2 + d2 = 1.Chứng minh 1 1 1 + + + + + ≤8 − ab − bc − cd − da − db − ca Ξ Bài tập 24 (Phạm Kim Hùng) Cho số thực không âm a, b, c, d, k có tổng Chứng minh (abc)k + (bcd)k + (cda)k + (dab)k ≤ max{4, ( )3k } Ξ Bài tập 25 (Phan Thành Nam) Cho số thực x, y, z, t thỏa: max{xy, yz, zt, tx} ≥ Chứng minh √ √ − xy + y + − yz + z + − zt + t2 + − tx + x2 ≥ 16 + (x − y + z − t)2 Ψ Bài tập 26 (Phan Thành Nam) Cho số thực x, y, z, t ∈ [−1, 1] thỏa mãn x + y + z + t = Chứng minh √ √ + x + y + + y + z + + z + t + + t + x2 ≥ (*Ghi chú: Bài xuất phát từ trường hợp ba số 20a, dó nhiên khó nhiều BĐT tương tự với số không nữa) Φ Bài tập 27 (Vasile Cirtoaje) Chứng minh a1, a2 , , an không âm có tổng n (n − 1)(a21 + a22 + + a2n ) + na1 a2 an ≥ n2 Ξ Bài tập 28: (Phạm Kim Hùng) Giả sử a1, a2, , an số thực không âm có tổng n Tìm gtnn biểu thức S = a21 + a22 + + a2n + a1a2 an( 59 1 + + + ) a1 a2 an Ξ baøi tập 29 Tìm số dương km tốt để BĐT sau với dãy số thực không âm x1 , x2, , xn có tổng n (1 + mx1)(1 + mx2) (1 + mxn ) ≤ (m + 1)n + km (x1 x2 xn − 1) m số dương Bài toán 30 (Phan Thành Việt) Cho a1 , a2, , an, s, k số thực dương n thỏa mãn: a1a2 an = sn n − = (1+s) k Xét BĐT: 1 + + + ≤n−1 k k (1 + a1) (1 + a2) (1 + an )k Θ a) Chứng minh BĐT nói chung không Φ b) (VMO 1999) Chứng minh BĐT trường hợp k = Ψ c) Tìm tất giá trò k (tùy thuộc n) để BĐT 60 ... đầu, đặc biệt BĐT biến cách khảo sát hàm biến số ta chứng minh BĐT đơn giản Chính tư tưởng giảm dần số biến nên phương pháp gọi phương pháp dồn biến Bây trình bày kó thuật phương pháp thông qua... Iran 1996 giải kó thuật dồn hai biến Từ có hai câu hỏi tự nhiên là, thứ nhất: toán giải cách dồn hai biến không, thứ hai: BĐT Iran 1996 giải cách dồn biến biên không? Chúng đề nghò bạn tự giải đáp... xảy có biến có giá trò biên (và ta gọi "cực trò đạt biên") Phương pháp dồn biến đặt để giải BĐT có dạng Ý tưởng chung là: ta đưa trường hợp có hai biến nhau, biến có giá trò biên, số biến giảm

Ngày đăng: 14/06/2020, 16:48

Từ khóa liên quan

Tài liệu cùng người dùng

Tài liệu liên quan